PT36.S1.Q4-- Help -- Most Strongly Supported (or Must Be True?) Question

DarklordDarklord Alum Member
edited May 2020 in Logical Reasoning 586 karma

Hi,

So this was a tough question because it is a MSS (Most Strongly Supported) question, but I was not able to really think of how the one right answer (C) must be true based on the information provided. Some tutors think that this question is just an example of how sometimes MSS answers are simply what is "most strongly supported" (and thus not necessarily must-be-true), but can anybody think of a way to make C must be true?

Thank you!
Any thoughts would be very much appreciated!

Admin note: https://7sage.com/lsat_explanations/lsat-36-section-1-question-04/

Comments

  • canihazJDcanihazJD Alum Member Sage
    edited May 2020 8313 karma

    I zeroed in on "most drugs cause weight gain" and "some weight gain is unlikely to be preventable" both indicating there was at least one instance of weight gain. That and none of the other ACs seemed like contenders.

    Most cause weight gain means at least some cause weight gain. But maybe you know most cause weight gain through means other than someone actually gaining weight... I suppose that's why it's a MSS instead of a MBT?

Sign In or Register to comment.